The third paragraph of the passage provides the most support for which one of the following inferences?

Shula on October 19 at 03:17AM

Why is E correct?

Hi, could you please explain why E is correct? I don't understand why the 3rd paragraph most supports E. Thank you so much!

Reply
Create a free account to read and take part in forum discussions.

Already have an account? log in

Emil-Kunkin on October 19 at 10:01PM

This is directly supported by lines 34-36